Física, perguntado por le0z1nh0, 8 meses atrás

Me ajuda por favor, preciso da resolução!!!!!!

Anexos:

Soluções para a tarefa

Respondido por andrewfairbairn
0

Resposta:

Esta não é fácil, amigo

Temos

m = 10kg

Θ1 = 30°

Θ2 = 45°

As tensões nas cordas T1 e T2

Se traçar uma linha reta do ponto da junção e até aonde a corda está amarrada a esquerda, podemos chamar esta de T1x, o mesmo da junção a direita, chamamos esta de T2x. Devemos somar estas forças.

ΣFx = 0 = T2 cos Θ2 - t1 cos Θ

Para as forças verticais, T1y para cima.

ΣFy = 0 = T1 sen Θ1 + T2 sen Θ2 - m·g

T2x = T2 cos θ

T2y = T2 sen Θ

T1 cos Θ1 = T2 cos Θ2

T1 = T2 ( cos Θ2 / cos Θ1)

T1 = (0,707 / 0,866) ( T2)

0 = 0,8164 · sen 30°  + T2 sen 45° - (10kg)(10m/s²)

1,115 · T2 = 100N

T2 = 100N / 1,115

T2 = 89,68N

T1 = (0,707 / 0,866) ( 89,68)

T1 = 73,2N

Explicação:

Respondido por MSGamgee85
4

Esse exercício é sobre equilíbrio estático.

A condição de equilíbrio para um ponto material é que a soma de todas as forças deve ser nula:

\boxed{\displaystyle\sum\vec{\mathsf{F}}=\mathsf{0}}}

Vou analisar o equilíbrio nos dois eixos: horizontal (x) e vertical (y). Pela figura abaixo, obtemos:

  • Equilíbrio Horizontal

\mathsf{T_{2x}=T_{3x}}\\\\\mathsf{T_2\cdot cos(30^o)=T_3\cdot cos(45^o)}\qquad \mathsf{(1)}

  • Equilíbrio Vertical:

\mathsf{T_1=P}\qquad \mathsf{(2)}\\\\\mathsf{T_1=M\cdot g}\\\\\mathsf{T_1=10\cdot10}\\\\\therefore \boxed{\mathsf{T_1=100\,N}}

\mathsf{T_1=T_{2y}+T_{3y}}

\mathsf{100=T_2\cdot sen(30^o)+T_3\cdot sen(45^o)}\qquad\mathsf{(3)}

  • Como sen (45º) = cos (45º), a equação ( 1 ) fica:

\\\mathsf{T_2\cdot cos(30^o)=T_3\cdot sen(45^o)}\qquad \mathsf{(4)}

  • Substituindo (4) em (3), obtemos:

\mathsf{100=T_2\cdot sen(30^o)+T_2\cdot cos(30^o)}\\\\\mathsf{100=T_2 \cdot \dfrac{1}{2}+T_2\cdot \dfrac{\sqrt{3}}{2}}\\\\\mathsf{200=T_2+T_2\cdot \sqrt{3}}\\\\\mathsf{T_2=\dfrac{200}{1+\sqrt{3}}}\\\\\\\therefore \boxed{\mathsf{T_2\approx73,3\,N}}

  • Portanto, substituindo esse valor na equação (4), obtemos:

\mathsf{T_3\cdot sen(45^o)=T_2\cdot cos(30^o)}\\\\\mathsf{T_3\cdot \dfrac{\sqrt{2}}{2}=(73,3)\cdot \dfrac{\sqrt{3}}{2}}}\\\\\mathsf{T_3=\dfrac{(73,3)\cdot \sqrt{6}}{2}}\\\\\therefore \boxed{\mathsf{T_3\approx89,6\,N}}

Obs.: verifique se o gabarito está certo, pois obtive resposta trocada para as tensões dos fios 2 e 3. Caso essa solução não te agrade, deixe um comentário para que possamos corrigir. =]

Continue aprendendo com o link abaixo:

Equilíbrio corpo extenso

https://brainly.com.br/tarefa/28197549

Bons estudos! =D

Equipe Brainly

Anexos:
Perguntas interessantes